[Rozgrzewka OM][MIX][Nierówności] Nierówności

Zadania z kółek matematycznych lub obozów przygotowujących do OM. Problemy z minionych olimpiad i konkursów matematycznych.
Regulamin forum
Wszystkie tematy znajdujące się w tym dziale powinny być tagowane tj. posiadać przedrostek postaci [Nierówności], [Planimetria], itp.. Temat może posiadać wiele różnych tagów. Nazwa tematu nie może składać się z samych tagów.
blublu
Użytkownik
Użytkownik
Posty: 3
Rejestracja: 13 lis 2020, o 16:27
Płeć: Mężczyzna
wiek: 20
Podziękował: 1 raz
Pomógł: 1 raz

Re: [Rozgrzewka OM][MIX][Nierówności] Nierówności

Post autor: blublu »

Wspaniale, można kontynuować.
Awatar użytkownika
Premislav
Użytkownik
Użytkownik
Posty: 15687
Rejestracja: 17 sie 2012, o 13:12
Płeć: Mężczyzna
Lokalizacja: Warszawa
Podziękował: 196 razy
Pomógł: 5220 razy

Re: [Rozgrzewka OM][MIX][Nierówności] Nierówności

Post autor: Premislav »

Niech \(\displaystyle{ n\in \NN^{+}, x\in \left(0, \frac{\pi}{2n}\right)}\). Proszę wykazać, że
\(\displaystyle{ \sum_{k=1}^{n}\frac{\sin((k+1)x)}{\sin(kx)}< 2\frac{\cos x}{\sin^{2}x}}\)
Awatar użytkownika
Premislav
Użytkownik
Użytkownik
Posty: 15687
Rejestracja: 17 sie 2012, o 13:12
Płeć: Mężczyzna
Lokalizacja: Warszawa
Podziękował: 196 razy
Pomógł: 5220 razy

Re: [Rozgrzewka OM][MIX][Nierówności] Nierówności

Post autor: Premislav »

Rocco Siffredi po zażyciu dodatkowej porcji viagry pisze:Za długo stoi…
To zadanie może nie jest typowe dla wątku (w którym rzadko pojawia się w zasadzie trygonometria, i trudno się dziwić). Jak można się dowiedzieć z imomath.com, jest to zadanie pierwsze z ukraińskiego TSE 1999 (część druga).
rozwiązanie:    
Nie pisałbym tego rozwiązania, gdyby udało mi się wymyślić bardziej eleganckie lub gdyby ktoś inny takie zaproponował, no ale nie stało się tak. :|

Nowe zadanie:
dla \(\displaystyle{ x,y,z\in \RR^{+}}\) proszę udowodnić nierówność
\(\displaystyle{ \left(1+\frac{x}{y}\right)\left(1+\frac{y}{z}\right)\left(1+\frac{z}{x}\right)\ge 2\left(1+\frac{x+y+z}{\sqrt[3]{xyz}}\right)}\)
H0t_Orange_B0i
Użytkownik
Użytkownik
Posty: 29
Rejestracja: 9 cze 2020, o 10:49
Płeć: Mężczyzna
wiek: 9
Podziękował: 9 razy
Pomógł: 4 razy

Re: [Rozgrzewka OM][MIX][Nierówności] Nierówności

Post autor: H0t_Orange_B0i »

Po rozpisaniu dostajemy
\(\displaystyle{ 1+ \frac{y}{z}+ \frac{x}{y}+ \frac{x}{z}+ \frac{z}{x}+ \frac{y}{x}+ \frac{z}{y}+1 \ge 2+ \frac{2(x+y+z)}{ \sqrt[3]{xyz} } }\)

Skracamy dwójkę
\(\displaystyle{ \frac{y}{z}+ \frac{x}{y}+ \frac{x}{z}+ \frac{z}{x}+ \frac{y}{x}+ \frac{z}{y}+ \ge \frac{2(x+y+z)}{ \sqrt[3]{xyz} } }\)

Duża liczba ułamków sugeruje użycie ciągów monotonicznych a ponieważ występuje wyrażenie to \(\displaystyle{ \sqrt[3]{xyz}}\) to podstawmy
\(\displaystyle{ x= a^{3} }\)
\(\displaystyle{ y= b^{3} }\)
\(\displaystyle{ z= c^{3} }\)
Dostajemy wówczas
\(\displaystyle{ \frac{ b^{3} }{ c^{3} }+ \frac{ a^{3} }{ b^{3} }+ \frac{ a^{3} }{ c^{3} }+ \frac{ c^{3} }{ a^{3} }+ \frac{ b^{3} }{ a^{3} }+ \frac{ c^{3} }{ b^{3} }+ \ge \frac{2( a^{3}+b^{3}+c^{3} )}{ abc } }\)

Zapiszmy to w bardziej przejrzystej formie

\(\displaystyle{ \left[\begin{array}{ccc} \frac{b^2}{c^2} & \frac{a^2}{b^2} & \frac{a^2}{c^2} & \frac{c^2}{a^2} & \frac{b^2}{a^2} & \frac{c^2}{b^2} \\ \frac{b}{c} & \frac{a}{b} & \frac{a}{c}& \frac{c}{a}& \frac{b}{a}& \frac{c}{b} \end{array}\right]}\) \(\displaystyle{ \ge}\) \(\displaystyle{ \left[\begin{array}{ccc} \frac{b^2}{c^2} & \frac{a^2}{b^2} & \frac{a^2}{c^2} & \frac{c^2}{a^2} & \frac{b^2}{a^2} & \frac{c^2}{b^2} \\ \frac{c}{a} & \frac{b}{c} & \frac{c}{b}& \frac{a}{b}& \frac{a}{c}& \frac{b}{a}\end{array}\right]}\)

Widzimy że po prawej stronie te dwa ciągi są zgodne monotonicznie więc dowolna permutacja dolnego ciągu z jaką mamy do czynienia po lewej stronie dowodzi prawdziwość nierówności dla \(\displaystyle{ a,b,c \in \mathbb{ R^{+} }}\)

Dodano po 1 godzinie 31 minutach 46 sekundach:
.
Awatar użytkownika
Premislav
Użytkownik
Użytkownik
Posty: 15687
Rejestracja: 17 sie 2012, o 13:12
Płeć: Mężczyzna
Lokalizacja: Warszawa
Podziękował: 196 razy
Pomógł: 5220 razy

Re: [Rozgrzewka OM][MIX][Nierówności] Nierówności

Post autor: Premislav »

Świetnie, Twoja kolej.
Ukryta treść:    
H0t_Orange_B0i
Użytkownik
Użytkownik
Posty: 29
Rejestracja: 9 cze 2020, o 10:49
Płeć: Mężczyzna
wiek: 9
Podziękował: 9 razy
Pomógł: 4 razy

Re: [Rozgrzewka OM][MIX][Nierówności] Nierówności

Post autor: H0t_Orange_B0i »

NIech \(\displaystyle{ x_{1}, x _{2} , x_{3} ...... x_{n}}\) należą do \(\displaystyle{ \mathbb R }\)

Udowodnić że zachodzi

\(\displaystyle{ \frac{ x_{1} }{1+ x_{1} ^{2} }+\frac{ x_{2} }{1+ x_{1} ^{2}+ x_{2} ^{2} }+.......+\frac{ x_{n} }{1+ x_{1} ^{2}+ x_{2} ^{2}+.....+ x_{n} ^{2} } < \sqrt{n} }\)
Awatar użytkownika
cmnstrnbnn
Użytkownik
Użytkownik
Posty: 84
Rejestracja: 4 mar 2019, o 20:45
Płeć: Mężczyzna
Lokalizacja: Lublin
Podziękował: 11 razy
Pomógł: 1 raz

Re: [Rozgrzewka OM][MIX][Nierówności] Nierówności

Post autor: cmnstrnbnn »

\(\displaystyle{ \frac{ x_{1} }{1+ x_{1} ^{2} }+\frac{ x_{2} }{1+ x_{1} ^{2}+ x_{2} ^{2} }+.......+\frac{ x_{n} }{1+ x_{1} ^{2}+ x_{2} ^{2}+.....+ x_{n} ^{2} } \le }\)
\(\displaystyle{ \frac{ x_{1} }{1+ x_{1} ^{2}+ x_{2} ^{2}+.....+ x_{n} ^{2} }+\frac{ x_{2} }{1+ x_{1} ^{2}+ x_{2} ^{2}+.....+ x_{n} ^{2} }+.......+\frac{ x_{n} }{1+ x_{1} ^{2}+ x_{2} ^{2}+.....+ x_{n} ^{2} }=\frac{ x_{1} + x_{2} +...+ x_{n} }{1+ x_{1} ^{2}+ x_{2} ^{2}+.....+ x_{n} ^{2} }}\)

Równość zachodzi, jeżeli \(\displaystyle{ x_{2} = x_{3} =....= x_{n}=0}\)

Z nierówności pomiędzy średnią kwadratową, a arytmetyczną
\(\displaystyle{ \sqrt{ \frac{ 1+ x_{1} ^{2}+ x_{2} ^{2}+.....+ x_{n} ^{2}}{n} } \ge \frac{1 + x_{1} + x_{2} +...+ x_{n} }{n} }\) więc
\(\displaystyle{ \sqrt{ n} \ge \frac{1 + x_{1} + x_{2} +...+ x_{n}}{ 1+ x_{1} ^{2}+ x_{2} ^{2}+.....+ x_{n} ^{2} } >\frac{ x_{1} + x_{2} +...+ x_{n} }{1+ x_{1} ^{2}+ x_{2} ^{2}+.....+ x_{n} ^{2} }}\)

co należało dowieść
Awatar użytkownika
Premislav
Użytkownik
Użytkownik
Posty: 15687
Rejestracja: 17 sie 2012, o 13:12
Płeć: Mężczyzna
Lokalizacja: Warszawa
Podziękował: 196 razy
Pomógł: 5220 razy

Re: [Rozgrzewka OM][MIX][Nierówności] Nierówności

Post autor: Premislav »

cmnstrnbnn, to nie zadziała, już pierwsza nierówność jest nieprawdziwa (dokładniej: nie zawsze prawdziwa), bo np. jeśli zwiększysz mianownik ułamka o dodatnim liczniku i mianowniku, to zmniejszysz ułamek, zamiast go zwiększyć.
rozwiązanie:    
Awatar użytkownika
cmnstrnbnn
Użytkownik
Użytkownik
Posty: 84
Rejestracja: 4 mar 2019, o 20:45
Płeć: Mężczyzna
Lokalizacja: Lublin
Podziękował: 11 razy
Pomógł: 1 raz

Re: [Rozgrzewka OM][MIX][Nierówności] Nierówności

Post autor: cmnstrnbnn »

Dobra racja, faktycznie bardzo głupi błąd popełniłem

To ode mnie w ramach zadośćuczynienia

\(\displaystyle{ a, b, c>0}\), oraz \(\displaystyle{ abc= \frac{1}{8} }\)

Udowodnij, że \(\displaystyle{ a^{2} + b^{2} + c^{2} + a^{2}b^{2} + a^{2}c^{2} + b^{2}c^{2} \ge \frac{15}{16} }\)
Awatar użytkownika
Premislav
Użytkownik
Użytkownik
Posty: 15687
Rejestracja: 17 sie 2012, o 13:12
Płeć: Mężczyzna
Lokalizacja: Warszawa
Podziękował: 196 razy
Pomógł: 5220 razy

Re: [Rozgrzewka OM][MIX][Nierówności] Nierówności

Post autor: Premislav »

Ukryta treść:    
Awatar użytkownika
cmnstrnbnn
Użytkownik
Użytkownik
Posty: 84
Rejestracja: 4 mar 2019, o 20:45
Płeć: Mężczyzna
Lokalizacja: Lublin
Podziękował: 11 razy
Pomógł: 1 raz

Re: [Rozgrzewka OM][MIX][Nierówności] Nierówności

Post autor: cmnstrnbnn »

Idealnie, można kontynuować zabawę
Awatar użytkownika
Premislav
Użytkownik
Użytkownik
Posty: 15687
Rejestracja: 17 sie 2012, o 13:12
Płeć: Mężczyzna
Lokalizacja: Warszawa
Podziękował: 196 razy
Pomógł: 5220 razy

Re: [Rozgrzewka OM][MIX][Nierówności] Nierówności

Post autor: Premislav »

Niech \(\displaystyle{ a_{1}, a_{2}\ldots a_{n}\in \RR^{+}}\) i niechaj \(\displaystyle{ s=\sum_{i=1}^{n}a_{i}}\). Proszę wykazać, że
\(\displaystyle{ \sum_{i=1}^{n}\frac{s+a_{i}}{s-a_{i}}\ge \left(\frac{n+1}{n-1}\right)^{2}\sum_{i=1}^{n}\frac{s-a_{i}}{s+a_{i}}}\)
bosa_Nike
Użytkownik
Użytkownik
Posty: 1665
Rejestracja: 16 cze 2006, o 15:40
Płeć: Kobieta
Podziękował: 71 razy
Pomógł: 445 razy

Re: [Rozgrzewka OM][MIX][Nierówności] Nierówności

Post autor: bosa_Nike »

Ukryta treść:    
Awatar użytkownika
Premislav
Użytkownik
Użytkownik
Posty: 15687
Rejestracja: 17 sie 2012, o 13:12
Płeć: Mężczyzna
Lokalizacja: Warszawa
Podziękował: 196 razy
Pomógł: 5220 razy

Re: [Rozgrzewka OM][MIX][Nierówności] Nierówności

Post autor: Premislav »

Ciekawe podejście (zresztą nie pierwszy raz tego typu rozwiązanie się pojawia w tym wątku), moje było trochę inne.
Ukryta treść:    
Oczywiście można kontynuować.
bosa_Nike
Użytkownik
Użytkownik
Posty: 1665
Rejestracja: 16 cze 2006, o 15:40
Płeć: Kobieta
Podziękował: 71 razy
Pomógł: 445 razy

Re: [Rozgrzewka OM][MIX][Nierówności] Nierówności

Post autor: bosa_Nike »

Dla rzeczywistych \(\displaystyle{ a,b,c,d\ge 0}\), takich że \(\displaystyle{ a+b+c+d=4}\), udowodnij $$\frac{a}{3a^3+2}+\frac{b}{3b^3+2}+\frac{c}{3c^3+2}+\frac{d}{3d^3+2}\le\frac{4}{5}.$$
ODPOWIEDZ